Módulo 3 Pediatría Flashcards

1
Q

A 4-week-old male infant has been spitting up his formula feedings for the past few days. He does not vomit bilious material or blood. The spitting up is gradually becoming more frequent, and forceful vomiting ensues. The vomitus seems to shoot straight out and nearly hit the wall. On examination, the baby seems hungry and is chewing his fist. His mucous membranes appear dry. A small, round mass, about the size of an adult thumbnail, is palpated in the upper abdomen. Laboratory data reveal Na+ of 133 mEq/L, K+ of 3.5 mEq/L, Cl− of 93 mEq/L, and HCO3 − of 29 mEq/L. Which of the following is the most appropriate next step in management?

(A) Change the feedings to clear liquids or Pedialyte
(B) Obtain a surgical consult immediately
(C) Obtain flat plate and upright x-ray films of the abdomen
(D) Insert a nasogastric tube
(E) Begin parenteral antibiotics

A

Respuesta: B

The correct answer is B. This infant has the classic presentation of pyloric stenosis. It occurs more commonly among first-born males, particularly white males. The cause of pyloric stenosis is unknown. It usually presents with nonbilious vomiting at 3-6 weeks of age. It progresses from intermittent vomiting to increasing numbers of episodes with more and more forceful vomiting. The infant acts hungry afterward, but as the vomiting continues there is loss of fluids and electrolytes resulting in hypochloremic metabolic alkalosis. Abdominal examination usually reveals a pyloric lump that feels like an olive-size mass. Treatment for pyloric stenosis is surgical; therefore, immediate consultation with the surgeon is necessary. The surgeon will preoperatively correct the metabolic alkalosis and surgically perform a pyloromyotomy.

Changing the feedings to clear liquids and a solution, such as Pedialyte (choice A), will not relieve this obstruction and will only delay the needed surgical treatment.

Obtaining abdominal x-ray films (choice C) will not delineate the specific problem. Appropriate x-ray films in this situation include either a barium swallow, which will reveal a large stomach with only thin streaks of barium in the pylorus due to the hypertrophied pylorus, or an ultrasound, which will reveal a doughnut appearance of the pyloric area.

Insertion of a nasogastric tube (choice D) will relieve an overdistended stomach, but will not treat the underlying problem.

Treating the infant with parenteral antibiotics (choice E) is not indicated. This is not an infectious condition, and prophylactic antibiotics are not used in conjunction with the surgery.

How well did you know this?
1
Not at all
2
3
4
5
Perfectly
2
Q

An infant has had repeated pneumonias and middle ear infections that began at about 5 months of age. At 1 year of age, serum electrophoresis demonstrated hypogam-maglobulinemia. T cell function was normal. By 2 years of age, the child’s infection rate has decreased, and repeat serum electrophoresis is normal. Which of the following immunoglobulins was likely decreased in this child during the period of increased susceptibility to infection?

(A) IgA
(B) IgD
(C) IgE
(D) IgG
(E) IgM

A

Respuesta: D

The correct answer is D. The condition is called transient hypogammaglobulinemia of infancy. It is due to exhaustion of maternally supplied IgG (the only antibody to cross the placenta in significant amounts) before the infant has begun to produce significant amounts of his own antibodies. Affected infants typically go through several months to years of being very vulnerable to infection, and then improve as their immune systems mature and they are able to produce more IgG. The condition does not present in the neonatal period because maternal IgG can cross the placenta during the third trimester of intrauterine life. The maternally supplied IgG is usually exhausted by about the 4th to 6th month of life.

IgA (choice A) is found in secretions and in serum but does not cross the placenta.

IgD (choice B) is a transiently produced antibody during B cell development that is present in trace amounts in serum.

IgE (choice C) is the antibody associated with allergic reactions; it does not cross the placenta.

IgM (choice E) is a large antibody found in serum that does not cross the placenta.

How well did you know this?
1
Not at all
2
3
4
5
Perfectly
3
Q

A 12-year-old, previously healthy girl presents to her physician with a chief complaint of early morning headaches. She states that these headaches wake her up from sleep 2 to 3 days a week. She also complains of some vomiting associated with the headaches. The headaches have been getting progressively worse for the past 2 months. She denies any photophobia, dizziness, or blurred vision. There is no history of a recent respiratory infection, runny nose, or cough. There is no history of recent trauma. In the office, her vital signs are within normal limits. Her examination shows pupils that are equal, round, and reactive, with no maxillary or frontal sinus tenderness. Her tympanic membranes are clear and intact. Her neck is supple with full range of motion. Neurologic examination shows a positive Romberg sign. Which of the following tests would most likely confirm the diagnosis?

(A) CT of the brain
(B) MRI of the brain
(C) Plain film of the skulli
(D) Sinus x-ray film
(E) Spinal tap

A

Respuesta: B

The correct answer is B. The early morning headaches associated with vomiting and getting progressively worse, along with the presence of a positive Romberg sign, suggest the presence of a space-occupying lesion (a brain tumor). An MRI of the brain would be the best diagnostic test to rule out brain tumors, posterior fossa tumors, which are of clinical concern in children.

CT of the head (choice A) would be a good first test in most cases, except when you are looking for a brain tumor in children because it may not show a posterior fossa tumor. For investigation of an intracranial bleed, a CT would be an appropriate test, although there is no recent history of trauma to suggest this diagnosis.

A plain skull x-ray film (choice C) would provide information regarding any fracture or lytic lesions, both unlikely in this example.

Sinus films (choice D) would be helpful in the diagnosis of sinusitis, which could relate to chronic headaches. However, this patient had vomiting with headaches, no sinus tenderness, and no history of chronic rhinitis, so sinusitis would be an unlikely diagnosis.

A spinal tap (choice E) would reveal increased intracranial pressure or an infection of the cerebrospinal fluid. However, it would not be the first test to do before ascertaining whether there was a spaceoccupying lesion in the brain.

How well did you know this?
1
Not at all
2
3
4
5
Perfectly
4
Q

A 10-month-old boy develops an upper respiratory tract infection 2 days before presentation. On the day of presentation, he has a generalized tonic-clonic seizure lasting 30 seconds. His temperature is 40.0 C (104.0 F), blood pressure is 90/60 mm Hg, and respirations are 22/min. He is alert and smiling. He has rhinorrhea, and his neck is supple. He has bruises below his knees. Which of the following is the most likely diagnosis?

(A) Child abuse
(B) Idiopathic epilepsy
(C) Infantile spasms
(D) Meningitis
(E) Simple febrile seizure

A

Respuesta: E

The correct answer is E. The most frequent cause of a short generalized tonic-clonic seizure is a simple febrile seizure. The patient must be treated with antipyretics, but prognosis is excellent. Febrile seizures are not considered to be epileptic since they are provoked events. This seizure type is seen typically in children aged 6 months to 3 years, but may occur until 6 years.

Given the high proportion of child abuse (choice A) cases seen in emergency departments, child abuse should always be on the differential. However, this patient’s history suggests other causes.

Idiopathic epilepsy (choice B) is also called the benign focal epilepsy of childhood. The age of onset is 4-10 years, and the disorder is not associated with underlying cerebral abnormalities.

Infantile spasms (choice C) are massive myoclonic seizures with either forceful flexion or extension of the trunk. The spasms occur in clusters, and the child may cry or be extremely irritable during these periods.

Meningitis (choice D) should be considered, but a lumbar puncture is not essential in this infant, who is alert and smiling and showing no meningeal signs on examination.

How well did you know this?
1
Not at all
2
3
4
5
Perfectly
5
Q

A previously well 3-year-old child presents to the clinic with marked erythema of the cheeks that appeared suddenly overnight. The rash has spread to involve the proximal arms, and it has a reticular erythematous maculopapular appearance. There are no known allergies, and the child has no prior illnesses and has not been exposed to any individuals who are ill. Vital signs are within normal limits. Cervical lymphadenopathy is noted on physical examination. Which of the following is the most likely cause of this child’s rash?

(A) Contact dermatitis
(B) Fifth disease
(C) Measles
(D) Roseola
(E) Rubella
(F) Systemic lupus erythematosus (SLE)

A

Respuesta: B

The correct answer is B. Fifth disease, or erythema infectiosum, is caused by infection with human parvovirus B19. The rash of erythema infectiosum starts on the cheeks, giving a “slapped cheek” appearance. The rash may disappear and recur.

Contact dermatitis (choice A) presents with a history of contact with an allergen. The skin is inflamed and itchy, with weeping and vesiculations. The site of involvement is often suggestive, e.g., the ear lobes and nape of neck (from nickel in jewelry), wrist (watch straps, bracelets), and feet (dyes in socks, tanning agents, glues in shoes). Typically, the dermatitis does not spread because the reaction is limited to the areas of contact.

The rash of measles (choice C) begins on the head and spreads down the body. It is erythematous and maculo-papular, becoming confluent with progression.

Roseola (choice D) is caused by human herpesvirus 6 and has an incubation period of 9 days. The rash is maculopapular and develops 3 to 4 days after fever.

Rubella (choice E) produces a maculopapular rash that has no prodrome in children. Lymphadenopathy develops. The rash starts on the face and then becomes generalized.

Systemic lupus erythematosus (SLE; choice F) is a multisystem autoimmune disease. An erythematous facial butterfly rash can result from sun exposure. Diagnosis requires symptoms from multiple organ systems (e.g., hematologic, joints, pulmonary, nervous system, renal, gastrointestinal).

How well did you know this?
1
Not at all
2
3
4
5
Perfectly
6
Q

A premature infant born at 28 weeks’ gestation develops respiratory distress shortly after birth. The infant is placed in an incubator with supplemental oxygen and is monitored in the neonatal intensive care unit. Given the severe prematurity of the infant, the physician gives strict instructions for the nursing staff regarding oxygen tension levels. Attempts at oxygen weaning are begun as soon as clinically feasible. Which of the following is characteristic of the eye damage that occurs following prolonged oxygen exposure in preterm neonates?

(A) Blood vessels in the vitreous
(B) Cotton wool exudates in the retina
(C) Microaneurysms of the retinal arterioles
(D) Papilledema of the optic nerve head
(E) Ulcers on the cornea

A

Respuesta: A

The correct answer is A. The physician is trying to prevent retinopathy of prematurity, also known as retrolental fibroplasia. This is a cause of permanent blindness in premature neonates with respiratory distress syndrome who are treated with high oxygen levels. The pathophysiology of this condition is related to the fact that the premature infant’s retinas are not yet fully vascularized, and the direction in which the vessels grow is partly in response to changes in oxygen tension. When the infant’s eyes are not protected from increased environmental oxygen levels, the vitreous of the eye develops a higher oxygen tension than the retina, and the blood vessels grow abnormally into the vitreous (where they block the light) rather than staying on the retina. Shielding the neonate’s eyes prevents this from happening and reduces the incidence of the complication of visual loss.

Cotton wool exudates in the retina (choice B) indicate vascular leakage, which is not a feature of this condition.

Microaneurysms of the retinal arterioles (choice C) are relatively specific for diabetic retinopathy, which is seen in adults rather than in neonates.

Papilledema of the optic nerve head (choice D) suggests increased intraorbital or intracranial pressure.

Ulcers on the cornea (choice E) are seen in gonorrhea ophthalmia and other serious conjunctival diseases.

How well did you know this?
1
Not at all
2
3
4
5
Perfectly
7
Q

A 7-year-old boy is brought to the clinic by his mother, who states that he has been complaining of abdominal pain for 2 to 3 days. He has been afebrile, with no vomiting or diarrhea. His mother states she brought him to the office today because she noticed a rash on his legs that is getting worse, and he is now complaining of knee pain. On examination, there are palpable purpuric lesions on both legs and buttocks. He has pain around his ankle and knee joints with minimal swelling, and no warmth or erythema. Which of the following is the most likely diagnosis?

(A) Dermatomyositis
(B) Gastroenteritis
(C) Henoch-Schönlein purpura
(D) Juvenile rheumatoid arthritis
(E) Kawasaki disease

A

Respuesta: C

The correct answer is C. Henoch-Schönlein purpura is a syndrome of small blood vessel vasculitis that presents as a triad of purpuric (“palpable purpura”) rash, intermittent abdominal pain, and arthritis. The rash usually affects the lower extremities, buttocks, and lower back. The abdominal pain is crampy, and there are often guaiac positive stools. The arthritis can be transient, most commonly affecting the knees and ankles.

Dermatomyositis (choice A) is an inflammation of striated muscle and skin. Most patients have fatigue, weight loss, anorexia, and fever. There is progressive weakness of proximal muscles.

Gastroenteritis (choice B) would be characterized by vomiting and diarrhea, often accompanied by fever. Gastroenteritis would not be associated with a purpuric rash or arthritis.

Juvenile rheumatoid arthritis (choice D) presents with polyarthritis. Fatigue, fever, weight loss, morning stiffness, and refusal to walk may be presenting complaints. It is unlikely to present with a purpuric rash and abdominal pain.

Kawasaki disease (choice E) is characterized by prolonged fever, bilateral nonpurulent conjunctivitis, cervical lymphadenopathy, fissuring of mucous membranes, desquamation of extremities, and truncal rash. None of these symptoms are present in this case.

How well did you know this?
1
Not at all
2
3
4
5
Perfectly
8
Q

A previously healthy 12-year-old boy is brought to the physician the day after a nocturnal crisis of difficulty breathing, chest tightness, and cough. He has a history of atopic dermatitis that resolved around 6 years of age. He now has no apparent respiratory distress. His breathing is regular, and his respirations are 12/min. Blood pressure, pulse, and temperature are normal. Chest examination reveals only a few crackles that quickly clear after coughing and mild end-expiratory wheezes. Which of the following is the most appropriate next step in diagnosis?

(A) Arterial blood gas analysis
(B) Bronchial provocation test with histamine or methacholine
(C) Complete blood count
(D) Chest x-ray examination
(E) Spirometry before and after administration of a bronchodilator

A

Respuesta: E

The correct answer is E. The chest examination may be normal or nearly normal between attacks in patients with mild asthma. A positive personal or family history of other allergic disorders (such as atopic dermatitis in childhood) is frequently found in those with allergic asthma. To confirm a clinical diagnosis, spirometric tests are particularly useful. These should be performed before and after administration of a short-acting bronchodilator to demonstrate that there is airflow obstruction and that this is promptly reversible. Airflow obstruction is supported by finding a forced expiratory volume in 1 second/forced vital capacity (FEV1 /FVC) ratio of <75%.

Arterial blood gas analysis (choice A) is not informative in the diagnosis of asthma, and the results may even be normal during mild exacerbations.

A bronchial provocation test with histamine or methacholine (choice B) may be used if spirometry is not diagnostic and asthma is strongly suspected. It must be performed in a specialized setting.

A complete blood count (choice C) may reveal blood eosinophilia, which is frequently present in chronic allergic disorders but would not support the specific diagnosis of asthma.

Chest x-ray examination (choice D) is necessary only when other disorders that mimic asthma, such as pulmonary infections or pneumothorax, are likely

How well did you know this?
1
Not at all
2
3
4
5
Perfectly
9
Q

A 7-year-old boy is brought to the physician because of persistent nasal obstruction for 6 months. There is no personal or family history of allergic disorders. Examination of the nasal fossae reveals bilateral ethmoidal polyps that protrude into the middle meatus and nasal cavity. Which of the following is the most appropriate next step in diagnosis?

(A) Cutaneous allergen testing
(B) Excisional biopsy
(C) Nasal provocation testing
(D) Pilocarpine iontophoresis sweat test
(E) Radioallergosorbent test (RAST)

A

Respuesta: D

The correct answer is D. Nasal polyps are not neoplasms, but rather a florid hyperplastic response of nasal and paranasal mucosa to chronic inflammation. Allergic rhinitis/sinusitis is the most common underlying condition. However, presence of polyps in children should raise the possibility of cystic fibrosis. This should be promptly investigated by a pilocarpine iontophoresis sweat test, which would show elevated sweat chloride in patients with cystic fibrosis. Up to 20% of cystic fibrosis patients ultimately develop nasal polyps.

Cutaneous allergen testing (choice A) and detection of allergenspecific serum IgE by in vitro RAST (choice E) constitute the proper course of action when there is clinical evidence of an allergic nature of nasal polyps. These methods allow identification of triggering allergen(s) and subsequent institution of appropriate therapy, i.e., allergen avoidance and/or desensitization treatments.

Excisional biopsy (choice B) is necessary only when nasal polyps have not regressed with pharmacologic therapies, namely topical application of steroids (beclomethasone) or a short course of oral prednisone. Histopathologic examination of removed nasal polyps usually confirms the allergic, inflammatory nature of the process by demonstrating chronic inflammatory infiltration and tissue eosinophilia.

Nasal provocation testing (choice C) is a direct allergen challenge performed by inhalation of allergens through the nose. This test may allow identification of involved allergens in case of a positive reaction.

How well did you know this?
1
Not at all
2
3
4
5
Perfectly
10
Q

A neonate is very small for gestational age and shows hypotonia and marked skeletal muscle and subcutaneous fat hypoplasia. During delivery, a large volume of amniotic fluid was released at rupture of membranes. The placenta was small, and only a single umbilical artery was noted. The face has a pinched appearance with hypoplastic orbital ridges, short palpebral fissures, and a small mouth and jaw. The head is small with prominence of the occiput. The ears are low set and malformed. The infant’s fists are clenched, with overlapping of the third and fourth fingers. The feet are clubbed, and the great toe is shortened. Which of the following is most likely diagnosis?

(A) 47,XXY
(B) Triple X
(C) Trisomy 13
(D) Trisomy 18
(E) Trisomy 21

A

Respuesta: D

The correct answer is D. This is trisomy 18, also known as Edwards syndrome and trisomy E. The phenotype described in the question stem is typical. Both club feet and rocker bottom feet are common. These infants often have multiple congenital anomalies, which may involve heart, lungs, diaphragm, abdominal wall, or urinary tract. Hernias and cryptorchidism may also occur. Most affected individuals die by 1 year of age; the few survivors usually show marked developmental delay.

47,XXY (choice A), or Klinefelter syndrome, causes males with tall stature and is usually not diagnosed in infancy.

Triple X (choice B) causes apparently phenotypically normal women and is usually not diagnosed in infancy.

Trisomy 13 (choice C), or Patau syndrome, produces many midline defects, including holoprosencephaly, cleft lip, cleft palate, microphthalmia with colobomas of the iris, scalp defects, and dermal sinuses; other defects are present as well.

Trisomy 21 (choice E), or Down syndrome, is characterized by laterally upward slanting eyes, Brushfield spots of the irises, simian creases, and cardiac anomalies.

How well did you know this?
1
Not at all
2
3
4
5
Perfectly
11
Q

A 2-year-old girl is taken to a pediatrician because she has developed a rash and seems unusually unsteady when she tries to walk. Physical examination demonstrates a diffuse rash on body parts exposed to sun. Also noted are short stature, possible mental retardation, and ataxia. Screening studies demonstrate increased total amino acids in the urine. Which of the following is the most likely diagnosis?

(A) Alkaptonuria
(B) Cystinuria
(C) Hartnup disease
(D) Fanconi syndrome
(E) Phenylketonuria

A

Respuesta: C

The correct answer is C. This is Hartnup disease, which is an autosomal recessive condition that produces a neutral aminoaciduria with increased renal clearance of alanine, asparagine, glutamine, histidine, isoleucine, leucine, phenylalanine, serine, threonine, tyrosine, valine, and tryptophan. This is accompanied by malabsorption of other amino acids, notably tryptophan, but also phenylalanine and methionine. The resulting tryptophan deficiency produces pellagra-like symptoms (tryptophan is used to synthesize nicotinamide) with photosensitive skin lesions, ataxia, and neuropsychiatric disturbances. Hartnup disease can be effectively treated (early diagnosis is important to limit neurologic manifestations) with good nutrition supplemented with nicotinamide.

Alkaptonuria (choice A) leads to urinary secretion of homogentisic acid (causing urine that turns black on standing) and causes arthritis and dark coloration of cartilage.

Cystinuria (choice B) principally produces urinary tract calculi.

Fanconi syndrome (choice D) produces a generalized dysfunction of the proximal tubules with glucosuria, generalized aminoaciduria, and hypophosphatemia (with bony abnormalities).

Phenylketonuria (choice E), which is characterized by excess phenylalanine in urine, is an important cause of mental retardation in young children. Most affected children have pale skin, blond hair, and blue eyes.

How well did you know this?
1
Not at all
2
3
4
5
Perfectly
12
Q

A 1-year-old child is brought in for a regular “well baby” checkup. The child appears to have strabismus. The reflection of a bright light from the ceiling of the examination room comes from a different place in each eye. The family explains that the child has always looked that way, and there has been no recent change in the appearance of his eyes. Which of the following is the most effective management?

(A) No treatment unless the condition has not resolved spontaneously by age 7
(B) Corrective lenses
(C) Each eye patched for a month at a time, alternating sides
(D) Surgical correction whenever he is old enough to decide whether he wants it for cosmetic reasons
(E) Surgical correction as soon as it is practical to do it

A

Respuesta: E

The correct answer is E. This is not an emergency, but strabismus must be corrected as soon as possible to enable the brain to learn to process images from both eyes. Otherwise, images from one side are suppressed; by age 7, permanent cortical blindness will occur in the suppressed side (amblyopia).

Spontaneous correction (choice A) does not occur. It is an oftenquoted myth, probably stemming from the fact that some children with a broad base of the nose appear to be cross-eyed and then “grow out of it.” They never had strabismus, and, if properly examined, the corneal reflection of a bright light would be seen coming from the same place in each eye.

Corrective lenses (choice B) are the treatment for a different type of strabismus: if a child has had normal alignment until he reaches an older age, and then gets “cross-eyed” whenever he focuses on a nearby object. This type of patient has an accommodation problem that can be corrected with appropriate lenses.

Eye patches (choice C) are used to force the brain to process images from the “bad eye” while preparations are made for surgical correction. Only the “good eye” is patched, and the notion of alternating patches would not be effective and could not be the only treatment.

Delayed correction for cosmetic purposes only (choice D) would be appropriate if the strabismus had not been diagnosed or treated during infancy, so that when the patient was first seen (after the age of 7), he already had irreversible cortical blindness.

How well did you know this?
1
Not at all
2
3
4
5
Perfectly
13
Q

A 6-year-old boy presents in clinic for a routine visit. Examination reveals coarse, dark pubic hair, an enlarged penis and testes, and acne of the face and upper back. His mother notes that he has a body odor similar to that of her teenage son after playing sports. The child is in the 99th percentile of height for his age group. Which of the following is the most likely diagnosis?

(A) Congenital adrenal hyperplasia
(B) Hypothalamic tumor
(C) Klinefelter syndrome
(D) Male pseudohermaphroditism
(E) XYY syndrome

A

Respuesta: B

The correct answer is B. The child is exhibiting isosexual precocious development. The enlarged gonads indicate that he has an increased exposure to gonadotropins, which may be the result of precocious release. Thus, increased testosterone is not the primary abnormality but is secondary to increased exposure to gonadotropins. In boys, a hypothalamic tumor can be found in an appreciable percentage of cases.

An ectopic source of androgenic hormones, such as congenital adrenal hyperplasia (choice A), causes gonadotropins to be shut off and the testes to be small.

Klinefelter syndrome (choice C) is a common cause of primary hypogonadism.

Male pseudohermaphroditism (choice D) refers to infants who are 46,XY males with testes but appear to have incomplete masculinization, including hypospadias. This can be the result of defects in testosterone synthesis, metabolism, or action at the cellular level.

The XYY syndrome (choice E) is associated with acne and tall stature but not with precocious puberty.

How well did you know this?
1
Not at all
2
3
4
5
Perfectly
14
Q

A 10-year-old girl is brought to a pediatrician because her mother notices that she stumbles frequently at night, even with adequate lighting. Visual field testing demonstrates a relatively narrow midperipheral ring scotoma. Ophthalmoscopy demonstrates dark pigmentation in a bone spicule configuration involving the equatorial retina. Additional findings include a waxy yellow appearance to the disk and narrowed retinal arteries. Which of the following is the most likely diagnosis?

(A) Cataract
(B) Central retinal artery occlusion
(C) Retinal detachment
(D) Retinitis pigmentosa
(E) Uveitis

A

Respuesta: D

The correct answer is D. Retinitis pigmentosa is an often hereditary progressive degenerative disease of the retina. Patients present with progressive night blindness, visual field constriction with a ring scotoma, and loss of acuity. Characteristically, the rods in the equatorial retina are affected first, producing a mid-peripheral ring scotoma that can be detected with visual field testing. The ophthalmoscopic findings illustrated in the question stem are typical; additional manifestations include cataracts, myopia, and opacities in the vitreous humor. Some patients also have hearing loss. The condition is slowly progressive (usually leading to complete blindness over a period of several decades) and is at the moment untreatable, although fetal retinal transplants show some promise.

A cataract (choice A) is an opacity of the lens of the eye.

Central retinal artery occlusion (choice B) causes sudden, painless, unilateral blindness, and is associated with pallor of the macula.

In the case of retinal detachment (choice C), ophthalmoscopy reveals a raised area in one retina and would not selectively affect night vision.

Uveitis (choice E) is an inflammation of the iris, ciliary body, or choroid, and may cause visual loss secondary to “floaters” in the vitreous humor.

How well did you know this?
1
Not at all
2
3
4
5
Perfectly
15
Q

A 5-year-old child develops left-sided ear pain, but her mother is too busy to take her to the pediatrician. There is no improvement in the child’s condition, and the mother has noticed painful, swollen, red areas behind the pinna. At this point, the child is brought to the emergency department, where the physician notes the presence of a creamy discharge in the left ear canal and a temperature of 38.6 C (101.5 F). Tuning fork tests localize hearing better on the left ear when placed on apex of head, and bone conduction is greater than air conduction in the left ear. Which of the following is the most likely diagnosis?

(A) Acute mastoiditis
(B) Barotitis media
(C) Cholesteatoma
(D) Chronic otitis media
(E) Meniérè disease
(F) Myringitis
(G) Otitis externa
(H) Secretory otitis media

A

Respuesta: A

The correct answer is A. This child has acute mastoiditis, which can complicate untreated (and occasionally treated) cases of acute otitis media, especially when caused by bacteria. This is a dangerous complication because the mastoid process is adjacent to the brain, and this destructive infection can spread to cause a purulent meningitis. CT scans can be very helpful in defining the extent of the infection. Initial treatment is with intravenous forms of the antibiotics used to cover acute otitis media; coverage is then switched after cultures demonstrate specific antibiotic sensitivities.

Barotitis media (choice B) is middle ear damage caused by rapid pressure changes, as in airplane descent or deep sea diving.

Cholesteatoma (choice C) is a benign condition in which stratified squamous keratinizing epithelium grows in the middle ear. It results in a smelly discharge from the ear, hearing loss, vertigo, headache, and facial nerve palsy. Local expansion may result in damage to adjacent structures, and the condition can be fatal if left untreated.

Chronic otitis media (choice D) is the term used for chronic ear drum perforation and the infection that accompanies it.

Ménière disease (choice E) causes the cluster of vertigo, tinnitus, and fluctuating hearing loss.

Myringitis (choice F) is inflammation of the tympanic membrane and is associated with Mycoplasma or viral respiratory infections. Myringitis bulbosa occurs when there are painful vesicles on the tympanic membrane. Herpes zoster of the eardrum presents with a similar condition.

Otitis externa (choice G) is a diffuse inflammation of the skin lining the external auditory meatus. There is either scanty discharge or no discharge. The condition is typically painful upon movement of the pinna.

Secretory otitis media (choice H) occurs when the eustachian tube is blocked and (initially noninfectious) serous fluid accumulates in the middle ear.

How well did you know this?
1
Not at all
2
3
4
5
Perfectly
16
Q

A 3-year-old girl is believed to have swallowed a marble. She presents to the emergency department unable to speak and begins to become cyanotic. Initial attempts at endotracheal intubation are unsuccessful. Which of the following is the most appropriate next step in management?

(A) Continued attempts at endotracheal intubation
(B) Cricothyroidotomy (surgical)
(C) Face mask 100% O2 with succinylcholine
(D) Formal tracheostomy
(E) Needle cricothyroidotomy

A

Respuesta: E

The correct answer is E. In any emergency situation, the first step is to secure an airway and verify breathing (the ABCs: airway, breathing, circulation). Because endotracheal intubation was unsuccessful, a surgical airway must be secured. There are two types of cricothyroidotomy: needle and surgical. Needle cricothyroidotomy is the procedure of choice in children younger than 12 years. A patient can be ventilated for 30 minutes this way, allowing time for a more secure airway. In addition, enough pressure may be generated to expel a foreign body in the glottic area.

Intubation (choice A) has already failed, and the patient is becoming cyanotic and needs oxygenation.

Surgical cricothyroidotomy (choice B) is reserved for patients older than 12 years.

A face mask with 100% O2 plus succinylcholine (choice C) is incorrect because there is complete obstruction, which must be bypassed for oxygenation to be successful.

Formal tracheostomy (choice D) is reserved for long-term management of the airway. In this acute setting, a rapid, temporary airway is essential.

How well did you know this?
1
Not at all
2
3
4
5
Perfectly
17
Q

A 24-month-old girl is brought to the pediatrician’s office for evaluation because her mother noticed a yellowish discharge on the girl’s underwear for the past 3 days. She had no fever, but her mother said she has been fussier recently. On physical examination, the girl is appears excessively anxious about contact with the physician. Her introitus is inflamed, and the hymeneal edge is jagged at the 8 o’clock position. A vaginal culture is taken. Which of the following organisms, if isolated from the vaginal vault, would constitute the most definitive evidence of sexual abuse?

(A) Candida albicans
(B) Chlamydia trachomatis
(C) Gardnerella vaginalis
(D) Pseudomonas aeruginosa
(E) Neisseria gonorrhoeae

A

Respuesta: E

The correct answer is E. The physical examination findings, in particular a posterior tear of the hymen, in this 24-month-old girl are highly suggestive of sexual abuse. However, physical findings alone are not enough to prove a sexual abuse case. Cultures taken from the vaginal vault of the suspected victim are essential in diagnosis and documentation. Of all the pathogens associated with sexual abuse, Neisseria gonorrhoeae provides definitive evidence.

Candida albicans (choice A) is a common infection of the diaper area for both boys and girls. Vaginal candidiasis can occur in a 24- month-old child.

Chlamydia trachomatis (choice B) can be vertically transmitted to the infant from the mother at birth, and persists for 3 years. It could be associated with sexual abuse but cannot be confirmatory in this case, when it is isolated from a 24-month-old girl.

Gardnerella vaginalis (choice C) presents as vaginitis with discharge. It is uncommon in young girls but does not confirm sexual abuse.

Pseudomonas aeruginosa (choice D) rarely causes vaginitis and is not associated with sexual abuse.

How well did you know this?
1
Not at all
2
3
4
5
Perfectly
18
Q

A premature neonate develops respiratory distress syndrome several hours after birth. The infant is placed on a respirator and given other appropriate care. However, when the infant reaches a corrected gestational age of 36 weeks, he does not tolerate weaning from the ventilator. A chest x-ray film demonstrates alternating areas of hyperaeration and pulmonary scarring, resulting in parenchymal streaks and hyperexpanded areas. Which of the following is the most likely diagnosis?

(A) Apnea of prematurity
(B) Bronchopulmonary dysplasia
(C) Cystic fibrosis
(D) Persistent pulmonary hypertension of the newborn
(E) Transient tachypnea of the newborn

A

Respuesta: B

The correct answer is B. This infant has bronchopulmonary dysplasia, which is an important chronic lung disorder that can complicate respiratory distress syndrome of the newborn. Damage to the lungs by mechanical ventilation produces alternating areas of emphysema and scarred lungs. The smooth muscles of bronchioles and arterioles may also be hypertrophied. The modern trend is to try to prevent this complication by using the smallest amount of mechanical ventilation that still adequately aerates the infant’s lungs. Management of significant degrees of bronchopulmonary dysplasia is problematic, and therapeutic regimens vary from neonatal unit to neonatal unit.

Apnea of prematurity (choice A) causes transient interruptions in breathing.

The lung problems of cystic fibrosis (choice C) are usually not present at birth, but develop after the child has repeated bouts of pneumonia.

Persistent pulmonary hypertension of the newborn (choice D) is due to a persistence of fetal type circulation (typically due to either a patent foramen ovale or a persistent patent ductus arteriosus), which clinically causes severe hypoxemia.

Transient tachypnea of the newborn (choice E) causes 2 or 3 days of difficult breathing in neonates who are slow to reabsorb amniotic fluid trapped in the lungs.

How well did you know this?
1
Not at all
2
3
4
5
Perfectly
19
Q

Approximately 19 days after having had a severe sore throat, a 10- year-old girl is taken to a pediatrician because she is complaining that her arms and legs hurt. The mother reports that before the extremity pain began, the child had a rash with irregular boundaries that lasted about a day. Physical examination demonstrates mild fever, as well as swelling and erythema around several large joints. Laboratory studies show an elevated erythrocyte sedimentation rate, and ECG demonstrates a prolonged PR interval. Which of the following is the most likely explanation for these findings?

(A) Antigenic mimicry
(B) Bacterial infection of valves
(C) Parasitic infection of myocytes
(D) Toxin production
(E) Viral infection of myocytes

A

Respuesta: A

The correct answer is A. The child has acute rheumatic fever. Jones criteria for diagnosing rheumatic fever includes using both “major” criteria (carditis, polyarthritis, chorea, erythema marginatum, subcutaneous nodules) and “minor” criteria (arthralgia, fever, erythrocyte sedimentation rate elevation, C-reactive protein elevation, prolonged PR interval). Either two major criteria or one major plus two minor criteria can be used for diagnosis. Typically, by the time the acute rheumatic fever develops, the preceding streptococcal sore throat has healed, and the patient is no longer infected. The damage that is produced is immunologically mediated, based on antigenic mimicry of bacterial antigens for human ones. A number of specific targets have been proposed, including Lancefield group A antigen mimicking mitral valve glycoprotein, part of the M protein antigen mimicking helical proteins such as myosin, protoplast membrane mimicking myocardial sarcolemma or neuronal tissue, and bacterial hyaluronate capsule mimicking hyaluronate-containing articular tissues.

Although streptococci, staphylococci, and other bacteria can infect cardiac valves (choice B), causing endocarditis, this would not be associated with arthritis and erythema marginatum.

Parasitic infection of myocytes (choice C) can occur with either trichinosis or Chagas disease. Neither of these conditions would present with sore throat followed by arthritis.

Toxic damage to the heart (choice D) can occur with diphtheria but would accompany the sore throat, not be delayed by weeks.

Viruses can cause myocarditis (choice E), but arthritis and erythema marginatum would not be present.

How well did you know this?
1
Not at all
2
3
4
5
Perfectly
20
Q

A 5-year-old boy is scheduled for a medical checkup and MMR booster before beginning the school year. All of his vaccinations are up to date, but his mother reports that he had a reaction to the first MMR shot, which was given at 12 months of age. The reaction involved a cough, red eyes, and a rash over the skin that resolved in about 1 week. The mother reports that the child is presently resolving from a “mild case of the flu,” with mild fever, coughing, and rhinorrhea, that has kept him awake during the last couple of nights. Past medical history is significant for anaphylaxis to previously administered amoxicillin and neomycin. There is no history of major illnesses. Which of the following is a contraindication to the administration of MMR vaccine in this patient?

(A) Anaphylaxis to amoxicillin
(B) Anaphylaxis to neomycin
(C) Pregnancy in the mother
(D) Previous febrile reaction to the MMR vaccine
(E) Upper respiratory tract infection with low-grade fever

A

Respuesta: B

The correct answer is B. It is important to know the contraindications to childhood immunizations. The true contraindications to the MMR vaccine include anaphylactic reaction to the vaccine or any of its components (neomycin, gelatin), immunodeficiency, pregnancy, and untreated and active tuberculosis. Precautions are given in cases of moderate or severe illness, recent administration of immunoglobulin containing blood products, and thrombocytopenia or thrombocytopenic purpura.

Anaphylaxis to amoxicillin (choice A) is not a contraindication to MMR vaccination because it is not a component of the vaccine. Components of the vaccination to which patients may be allergic include neomycin and gelatin.

MMR vaccination is contraindicated in pregnant women due to its possible effects on fetal development. However, MMR vaccination is not contraindicated in persons who are in contact with pregnant women (choice C).

Previous febrile reaction to the MMR vaccine (choice D) is not a contraindication to vaccination. Some patients can develop measleslike infection with cough, cold, red eyes, and rash. However, it typically lasts 2 to 5 days and is self-limiting. An anaphylactic reaction to a previous vaccination, however, is a contraindication to repeat doses.

Febrile respiratory illness or any other active febrile infection (choice E) is not a contraindication. However, the Advisory Committee on Immunization Practices (ACIP) for the Centers for Disease Control (CDC) has recommended that vaccines be administered to persons with minor illnesses, such as diarrhea, mild upper respiratory infection with or without low-grade fever, and other low-grade febrile illnesses.

How well did you know this?
1
Not at all
2
3
4
5
Perfectly
21
Q

A 15-year-old boy is seen in the pediatrician’s office for a health maintenance physical examination. The boy reports a heavy, dragging sensation in his left scrotum. The sensation is more pronounced after exercise. He denies any scrotal pain. He is not sexually active. Examination of his genitalia indicates Tanner stage 4. There is a palpable fullness over his left scrotum. Both testes are normal in size and smooth in contour. Which of the following is the most likely explanation of these findings?

(A) Hydrocele
(B) Inguinal hernia
(C) Orchitis
(D) Testicular tumor
(E) Varicocele

A

Respuesta: E

The correct answer is E. The findings in the history and physical examination are most consistent with the diagnosis of varicocele, which is an abnormally dilated pampiniform venous plexus. Varicocele is very common; it is found in 15 to 20% of male adolescents and is usually asymptomatic. Patients with symptomatic varicocele usually describe a heavy, dragging sensation in the groin or the scrotum on the affected side. Palpation along the spermatic cord often reveals the classic “bag of worms” mass above the testis. The size of the varicocele increases with Valsalva maneuver and decreases with lying down.

Hydrocele (choice A) is the presence of fluid within the tunica vaginalis. It may be noncommunicating, when the mass is confined to the scrotum, or communicating, when there is continuity from the tunica vaginalis to the peritoneum. Hydroceles are typically asymptomatic, although a large hydrocele may interfere with ambulation.

Inguinal hernia (choice B) usually presents with an increase in scrotal size. It occurs more commonly in males than in females. Surgical repair is indicated to prevent complications.

Orchitis (choice C) is inflammation of the testis. It can be caused by sexually transmitted diseases, such as gonorrhea or chlamydia, or by viruses, such as Epstein-Barr virus, influenza, varicella, or coxsackieviruses. Orchitis usually presents with a painful and enlarged testicle.

Testicular tumor (choice D) usually presents as a painless mass that is firm, hard, and inseparable from the testis.

22
Q

A 5-year-old child undergoes a school entrance physical examination. The pediatrician notices grey-brown pigmentation on the skin of his forehead, hands, and pretibial regions. Subconjunctival areas near the corneoscleral junction show wedgeshaped, yellow-brown discoloration (pingueculae). Enlargement of both the spleen and the liver are noted on abdominal examination. Needle biopsy of the spleen demonstrates the presence of unusually large (20- to 100-mm diameter) reticuloendothelial histiocytes with a “crumpled-silk” appearance. Bone marrow biopsy demonstrates the presence of the same type of cells. Which of the following is the most likely diagnosis?

(A) Abetalipoproteinemia
(B) Fabry disease
(C) Gaucher disease
(D) Niemann-Pick disease
(E) Tangier disease

A

Respuesta: C

The correct answer is C. This is Gaucher disease, which is a familial autosomal recessive disorder of lipid metabolism. The disease is caused by a lack of glucocerebrosidase activity, which normally hydrolyzes glucocerebroside to glucose and ceramide, and occurs in three major clinical forms. All three types are characterized by prominent splenomegaly and accumulation of abnormal glucocerebrosides in reticuloendothelial cells in many organs (spleen, liver, bone marrow, and brain in more severe cases), producing the pathognomonic “crumpled-silk” histiocytes. This case is an example of the type 1, or the adult chronic nonneuronopathic, form. Type 1 is the most common form and actually presents frequently in childhood, although the most serious manifestations are often deferred to adulthood. Type 1 has an increased frequency in Ashkenazi Jews and manifests with hypersplenism, splenomegaly, and bone lesions. The pingueculae and brown skin pigmentation noted in the question stem may be helpful clues on physical examination. Type 2, the acute infantile neuronopathic form, causes splenomegaly, severe neurologic abnormalities, and death in early childhood. Type 3 is a juvenile form and has features intermediate to types 1 and 2.

Abetalipoproteinemia (choice A) is characterized by absent betalipoproteins, steatorrhea, acanthocytes, retinitis pigmentosa, ataxia, and mental abnormalities.

Fabry disease (choice B) is characterized by angio-keratomas, corneal opacities, burning extremity pain, and involvement of kidneys, heart, and brain.

Niemann-Pick disease (choice D) may clinically resemble Gaucher disease, but is characterized by “sea-blue” rather than “crumpledsilk” histiocytes.

Tangier disease (choice E) may resemble Gaucher disease, with hepatosplenomegaly and neurologic disease, but a helpful distinctive feature is orange-yellow tonsillar hyperplasia

23
Q

A 12-year-old boy presents to his pediatrician with frequent episodes of headache, nausea, blurry vision, and sweating. On physical examination, his temperature is 37.4 C (99.3 F), blood pressure is 148/94 mm Hg, pulse is 92/min, and respirations are 18/min. The rest of his examination is unremarkable. His 24-hour urinary vanillylmandelic acid (VMA) and metanephrines are increased. An abdominal CT reveals an extrarenal mass above the left kidney. Which of the following is the most appropriate pharmacotherapy?

(A) Alpha-adrenergic blocker
(B) Angiotensin-converting enzyme inhibitor
(C) Beta-adrenergic blocker
(D) Calcium channel blocker
(E) Diuretics

A

Respuesta: A

The correct answer is A. The clinical presentation of this patient is consistent with pheochromocytoma; this impression is confirmed by the increased levels of urinary catecholamines and metabolites, and an extrarenal mass on abdominal CT. Pheochromocytoma arises from the chromaffin cells in the adrenal medulla or the abdominal sympathetic chain. Clinical manifestations include headache, sweating, nausea, vomiting, palpitation, blurry vision, nervousness, and hypertension. In fact, pheochromocytoma is one of the important causes of secondary hypertension. In a 12-year-old boy with a blood pressure of 148/94 mm Hg, the differential diagnosis of secondary hypertension should be triggered in every clinician’s mind. If pheochromocytoma is suspected, a 24-hour collection of urine for catecholamines and metabolites (vanillylmandelic acid and metanephrines) can be checked. If these substances are elevated, the diagnosis of pheochromocytoma is confirmed. CT scan of the abdomen may be done to locate the tumor. Most pheochromocytomas are benign. When hypertension is present, as in this clinical vignette, it should be treated promptly, even before surgical resection of the tumor. An alpha-adrenergic blocker, such as phenoxybenzamine, is the drug of choice because it has been shown to effectively decrease blood pressure in patients with pheochromocytoma.

Angiotensin-converting enzyme inhibitor (choice B) does not play any role in controlling blood pressure in children with pheochromocytoma.

A beta-adrenergic blocker (choice C) is actually contrain-dicated in the initial treatment of hypertension in pheochromocytoma because it causes an unopposed alpha effect that would result in a paradoxical rise of blood pressure. However, labetalol, which is a combined alpha- and beta-blocker, can be used in some patients with efficacy similar to phenoxybenzamine.

Calcium channel blockers (choice D) and diuretics (choice E) are not used for treatment of hypertension in patients with pheochromocytoma.

24
Q

A 12-year-old girl with mild asthma comes to the office for a health maintenance visit. Her mother states that she is using her albuterol inhaler 2 to 3 times a week and that she has a cough that wakes her up at night about 3 times a month. On physical examination, she has diffuse inspiratory and expiratory wheezes. She has no accessory muscle use. Pulse oximetry shows 95% oxygen saturation on room air. Which of the following is the most appropriate treatment for her at this time?

(A) Albuterol nebulized treatment
(B) Cromolyn sodium nebulized treatment
(C) Oxygen via nasal cannula
(D) IV steroids
(E) Subcutaneous epinephrine

A

Respuesta: A

The correct answer is A. This child has mild asthma, requiring an albuterol treatment but no other urgent treatment at this time. Albuterol is used as a “rescue” medicine for someone having an acute attack.

Cromolyn sodium (choice B) is used as a daily medicine to control asthma and has no benefit in situations requiring an immediate effect. It is not used in acute attacks.

Oxygen via nasal cannula (choice C) is unnecessary at this time since she has an oxygen saturation greater than 92%.

IV steroids (choice D) would not be necessary since the patient is experiencing only mild symptoms. If the patient were thought to need steroid treatment, oral steroids would suffice.

Subcutaneous epinephrine (choice E) is rarely needed except in lifethreatening situations, such as patients with severe asthma in status asthmaticus

25
The 1-year-old brother of a child with known abetalipoproteinemia is evaluated by a pediatrician for the disease. The 1-year-old has been exhibiting steatorrhea and ataxia. Which of the following would most strongly support the suspected diagnosis? (A) Acanthocytes on peripheral smear (B) “Crumpled silk” histiocytes on bone marrow biopsy (C) Globoid cells on brain biopsy (D) Metachromatic deposits on sural nerve biopsy (E) "Sea-blue” histiocytes on bone marrow biopsy
Respuesta: A The correct **answer is A**. Numerous acanthocytes (erythrocytes with spiny membrane projections) on peripheral smear are a feature of abetalipoproteinemia, which is caused by a mutation in the gene for microsomal triglyceride transfer protein. Abetalipoproteinemia is a rare, usually autosomal recessive, congenital disorder characterized by a complete absence of beta-lipoproteins, steatorrhea, blindness due to retinitis pigmentosa, ataxia, and mental retardation. The complete absence of betalipoproteins means that neither chylomicrons nor VLDL is formed. Massive-dose vitamin A and E therapy has delayed neurologic deterioration in some cases. "Crumpled silk” histiocytes on bone marrow biopsy (**choice B**) suggest Gaucher disease. Globoid cells on brain biopsy (**choice C**) suggest Krabbe disease. Metachromatic deposits on sural nerve biopsy (**choice D**) suggest metachromatic leukodystrophy. "Sea-blue” histiocytes on bone marrow biopsy (**choice E**) suggest Niemann-Pick disease.
26
A 6-year-old boy is brought to the emergency department because of the acute onset of headache, nausea, and vomiting. On arrival, physical examination reveals marked nuchal rigidity and funduscopic evidence of papilledema. A head CT scan reveals a solid tumor in the posterior fossa, centered in the cerebellar vermis and extending to the fourth ventricle. An emergency craniotomy is performed, during which a small sample of the tumor is sent to the pathologist for a frozen section consultation. Which of the following is the most likely diagnosis? (A) Ependymoma (B) Glioblastoma multiforme (GBM) (C) Hemangioblastoma (D) Medulloblastoma (E) Oligodendroglioma (F) Pilocytic astrocytoma
Respuesta: D The correct **answer is D**. The most common posterior fossa tumors in children include medulloblastoma, ependymoma, and pilocytic astrocytoma. Medullo-blastoma, an anaplastic neoplasm thought to derive from primitive neuroectodermal precursors, belongs to the group of primitive neuroectodermal tumors (PNET). It originates from the cerebellar vermis and grows into the fourth ventricle, often producing signs and symptoms of increased intracranial pressure. The histologic features are those of other small round cell tumors. Ependymoma (**choice A**) usually derives from the ependymal cells lining the central canal. Childhood ependymomas typically develop in the fourth ventricle and may present with increased intracranial pressure. In contrast to medulloblastoma, however, ependymoma appears on CT/MRI as a mass that fills the fourth ventricle and secondarily involves adjacent structures. Glioblastoma multiforme (GBM; **choice B**) is a rare tumor in children of this age, and the cerebellum is a rare location at any age. Furthermore, GBM grows in a diffuse fashion within the white matter without involving the ventricular cavities. Hemangioblastoma (**choice C**) is a vascular tumor of uncertain histologic origin. The cerebellar hemisphere is its most common location, and it often appears as a cystic mass with a mural nodule. Its association with von Hippel-Lindau syndrome is noteworthy. Oligodendroglioma (**choice E**) is a slow-growing tumor of adults that usually develops in the white matter of a cerebral hemisphere. Calcifications within the tumor are frequently detected on x-ray films or CT scan, and seizures are often the presenting sign. Pilocytic astrocytoma (**choice F**) is a low-grade, well-circumscribed astrocytoma of children and young adults. Cerebellar pilocytic astrocytoma often presents as a cyst with a mural nodule (like hemangioblastoma) within a cerebellar hemisphere. Surgical resection is usually curative.
27
A 3-month-old infant with a history of chronic constipation presents with fulminant, watery diarrhea over 2 days. The infant is brought to the emergency department with signs of severe dehydration and a distended abdomen. His history is significant for delayed passage of meconium after birth and failure to thrive. Plain x-ray films of the abdomen reveal a massively dilated transverse colon, with the presence of gas in the distal colon. Which of the following is the most likely diagnosis? (A) Congenital hypothyroidism (B) Meconium ileus (C) Necrotizing enterocolitis (D) Toxic enterocolitis (E) Volvulus
Respuesta: D The correct **answer is D**. The most likely cause of this infant’s presentation is toxic enterocolitis (toxic megacolon), which is most often seen in the setting of Hirschsprung disease. The disease is suggested in this infant by the delayed passage of meconium, chronic constipation, and the failure to thrive. When unrecognized, putrefaction and development of colitis can occur. In its severest form, it presents as toxic megacolon. The resultant diarrhea, with potentially fatal fluid loss, is due to bacterial overgrowth in the dilated bowel with resulting production of bacterial toxins. Although fluid replacement and antibiotics are important in the management of the these cases, the most helpful therapeutic intervention is prompt surgical resection of the involved bowel with formation of a colostomy. Infants with congenital hypothyroidism (**choice A**) have a history of constipation. However, other features of hypothyroidism are lacking in this case. Classic features include prolonged jaundice, umbilical hernia, macroglossia, hypotonia, and failure of the posterior fontanelle to close. Meconium ileus (**choice B**) occurs in infants with cystic fibrosis who fail to pass meconium in the first few days of life. Necrotizing enterocolitis (**choice C**) is a feared complication of the newborn period, with abdominal distension and sepsis. Volvulus neonatorum (**choice E**) is usually due to a defect in the normal rotation of the bowel in which the caecum remains high, often with a congenital band passing across the duodenum. This results in an intestinal obstruction and vessel occlusion at the base of the mesentery. Clinically, the features of intestinal obstruction predominate, with vomiting, abdominal pain, and constipation. Abdominal radiograph shows a “comma sign” or “beak sign” with absence of distal bowel gas.
28
A mother brings her 9-month-old daughter to the pediatrician with complaints of a rash. The mother states that the infant had a high fever (temperature up to 40.0 C [104.0 F]) for 3 days prior to developing the rash, but is now afebrile. The mother also says that the infant has had a runny nose and a slight cough for the past 3 days. On examination, there is a fine macular rash on the infant’s trunk and neck. The examination is otherwise within normal limits, and the infant is playful and smiling. Which of the following is the most likely diagnosis? (A) Erythema infectiosum (B) Roseola (C) Rubella (D) Rubeola (E) Varicella
Respuesta: B The correct **answer is B**. Roseola is a virus that causes an exanthem (exanthem subitum). It typically affects children between 6 months and 3 years of age. Most cases manifest with a fever (temperature often up to 40-40.5 C) that lasts for 2-3 days; there is often mild erythema of the pharynx and congestion. The discrete, macular rash typically erupts on the trunk with an abrupt end to the fever. Erythema infectiosum (**choice A**), also known as fifth disease, is caused by parvovirus B19. This virus usually affects children aged 5-14 years. There is usually a nonspecific febrile illness with malaise and myalgias lasting 2-3 days. The rash of erythema infectiosum is characterized by a red rash on the cheeks (“slapped” cheeks) and a erythematous maculopapular rash on the extremities and trunk described as a “lacy” reticular pattern. Rubella (**choice C**), also known as German measles, is characterized by a low-grade fever (temperature usually less than 39 C), erythematous maculopapular rash, and lymphadenopathy. Rubeola (**choice D**), or measles, is characterized by 3-5 days of increasing fever, coryza, cough, and conjunctivitis. Koplik spots (white lesions on a red mucosa) appear in the mouth. There is a discrete, erythematous, maculopapular rash that begins on the face and moves downward, covering the trunk and upper and lower extremities. Varicella (**choice E**), or chickenpox, begins with a period of malaise and low-grade fever. The lesions begin as macules and progress to fluid filled vesicles that then crust; the crust eventually falls off. The fever is usually gone by the end of the first week of the rash.
29
An infant is born with a craniofacial abnormality. There is a patch of very white skin on the midline forehead, above which is a white forelock of hair. The remainder of the hair is dark brown. The eyes are also abnormal, with color variations from brown to blue within the irises. The medial canthi are displaced laterally. The infant does not respond to sound. Which of the following is the most likely diagnosis? (A) Beckwith-Wiedermann syndrome (B) Fragile X syndrome (C) Goldenhar syndrome (D) Pierre Robin syndrome (E) Treacher Collins syndrome (F) Waardenburg syndrome
Respuesta: F The correct **answer is F**. The child most likely has Waardenburg syndrome, which is one of the more common craniofacial abnormalities. Defects of this type are related to maldevelopment of the first and second vertebral arches that form the face. The defects usually begin during the second month of gestation. The features illustrated in the question stem are typical. On a practical basis, you should remember that children with white forelocks (partial albinism) should be tested for hearing loss. Beckwith-Wiedermann syndrome (**choice A**) is characterized by macroglossia, a large fontanelle, and a linear fissure in the external ear. Organomegaly of both the pancreas and kidney is common, and omphalocele may be seen at birth. Clinical features of fragile X syndrome (**choice B**) include mental retardation, large testicles, a prominent jaw, large ears, and a large head. The syndrome is caused by a dominant X-linked trinucleotide repeat gene with only a 50% penetrance in females. Features are most prominent in affected males. Goldenhar syndrome (**choice C**) produces an asymmetric face, malformed ears, small eyes, hearing loss, a large mouth, a small mandible, and vertebral anomalies. Pierre Robin syndrome (**choice D**) produces micrognathia, glossoptosis, and cleft palate. Treacher Collins syndrome (**choice E**) is characterized by malformation of the external ear, deafness, micrognathia, antimongoloid eye slanting, and coloboma of the lower eyelids.
30
A 15-month-old boy is brought into the clinic with a 1-day history of fever, decreased oral intake, and runny nose. His vital signs are within normal limits except for a temperature of 39.5 C (103.1 F). He is active and in no distress. His ears are clear bilaterally. There is a clear nasal discharge. Multiple small (1 to 2 mm) vesicular lesions are noted on the mucosa of the anterior tonsillar pillars and posterior palate. The rest of his examination is within normal limits. Which of the following is the most likely diagnosis? (A) Aphthous stomatitis (B) Hand-foot-and-mouth disease (C) Herpangina (D) Kawasaki disease (E) Stevens-Johnson syndrome
Respuesta: C The correct **answer is C**. Herpangina is a caused by coxsackievirus and usually manifests with dysphagia and vesicles on the anterior tonsils and palate. There is typically a history of fever and viral symptoms (runny nose, cough). Diagnosis is made by the presence of small vesicular lesions on the anterior tonsils and posterior palate. Aphthous stomatitis (**choice A**) usually presents with ulcers on the buccal mucosa. There is no associated fever, and the patient does not usually complain of pain. Hand-foot-and-mouth disease (**choice B**) is also caused by coxsackievirus and presents with small vesicles in the mouth, but there are also vesicles on the hands and feet. Commonly, a maculopapular rash is also present. Kawasaki disease (**choice D**) is associated with a history of prolonged fever (more than 5 days), conjunctival erythema, cracked lips, maculopapular rash, cervical lymphadenopathy, and swollen hands. Stevens-Johnson syndrome (**choice E**) is a systemic disease that presents with ulcers over the entire mouth and lips. There usually is a confluent rash on the body as well. The child will appear very ill
31
A nurse notices that a 1-week-old, premature infant in the neonatal unit is experiencing migratory jerks of the extremities. She picks the infant up and can feel that the muscle jerks continue to happen, even when she holds an involved extremity still. After about 5 minutes, the jerking movements stop. Which of the following is the most appropriate first step in diagnosis? (A) CT scan of head (B) Electroencephalography (C) Serum chemistries (D) Skull x-rays (E) Ultrasound of head
Respuesta: C The correct **answer is C**. The infant is having neonatal seizures, which can take a variety of forms. In addition to the migratory clonic jerks described in the question stem, these infants may alternatively experience alternating hemiseizures or primitive subcortical seizures. The later may take the form of respiratory arrest, chewing motions, eye deviations, or changes in muscle tone. True grand mal seizures are usually not seen, possibly because the immaturity of the brain makes it difficult to transmit a seizure to all sites. Neonatal seizures can be seen in a variety of settings, which can be roughly subclassified as primary CNS versus metabolic. The metabolic causes are common, and screening chemistry studies can identify potentially correctable hypoglycemia, hypocalcemia, hypomagnesemia, hypernatremia, or hyponatremia. Consequently, the earliest diagnostic step is to order these appropriate serum screening tests, usually on a STAT basis. CT scan of head (**choice A**), EEG (**choice B**), skull x-rays (**choice D**), and ultrasound of the head (**choice E**) can all be very useful in offering diagnostic clues to true CNS problems, but should be deferred until after the child’s metabolic status is established.
32
A female neonate is undergoing an examination after birth. She was born to a 33-year-old primigravid mother at term via a normal spontaneous vaginal delivery. The pregnancy was uncomplicated, except for a positive maternal group B Streptococcus culture at 36 weeks’ gestation, for which the mother received penicillin during labor. The infant’s APGAR scores are 8 at 1 minute and 9 at 5 minutes. The mother notices that the infant has prominent labia and a dull pink vaginal epithelium. Which of the following is the most likely cause of the appearance of the infant’s genitalia? (A) Exposure to maternal estrogen (B) Exposure to penicillin (C) Infection with Chlamydia (D) Infection with group B Streptococcus (E) Sexual abuse
Respuesta: A The correct **answer is A**. At birth, a neonate will typically exhibit some signs of exposure to maternal estrogen. Two of these signs include prominent labia and a dull pink vaginal epithelium. As estrogen levels fall, the labia will become less prominent and the vagina will take on a brighter red appearance. In the absence of other signs or symptoms of disease, these findings are normal and do not require further evaluation. Exposure to penicillin (**choice B**) will almost never cause findings isolated to the labia and vagina. Infection with Chlamydia (**choice C**) can lead to neonatal pneumonia and eye infection. Infection with group B Streptococcus (**choice D**) can cause respiratory distress, pneumonia, meningitis, and sepsis in the newborn. It does not typically cause findings isolated to the genitalia. Sexual abuse (**choice E**) almost always must be considered whenever there are complaints or findings regarding an infant’s genitalia. In this case, however, the infant is newborn-only minutes old. Therefore, sexual abuse would not be in the differential diagnosis.
33
A 3-year-old girl is brought to the pediatrician with complaints of abdominal pain and fever. Her mother states that the fever started 2 days ago, with the highest temperature being 39.0 C (102.2 F). She has had no vomiting or diarrhea. The mother states that her daughter has been complaining of pain on urination. On examination, she is tender in her lower abdomen, and there is some right-sided costovertebral angle tenderness. A urinalysis confirms the suspicion of a urinary tract infection. Which of the following would be the most appropriate diagnostic procedure? (A) Cystoscopy (B) Dimercaptosuccinic acid (DMSA) scan in 1 to 2 months (C) Intravenous pyelogram (D) Voiding cystourethrogram (VCUG) now (E) VCUG in 1 to 2 months
Respuesta: E The correct **answer is E**. A voiding cystourethrogram (VCUG) is the appropriate test to run to evaluate the lower urinary tract and the presence of any reflux of urine into the kidney. It is appropriate to first wait until the acute infection is over, since there may be some degree of reflux present during an infection. The usual procedure is to wait 1-2 months after treatment to perform the VCUG to evaluate for reflux. Cystoscopy (**choice A**) is not indicated in the evaluation of urinary tract infection. Dimercaptosuccinic acid (DMSA) scan (**choice B**) is a radionuclide renal imaging scan. It might be an appropriate test to diagnosis acute pyelonephritis, but it is not indicated as a first test in evaluating reflux in a urinary tract infection 2 months after the infection. It is the most accurate test to diagnose acute pyelonephritis. Intravenous pyelogram (**choice C**) is rarely used in the pediatric population to evaluate for a urinary tract infection. Performing a VCUG now (**choice D**) would not be appropriate since there might be some degree of reflux present during an acute infection.
34
A neonate has an obviously abnormal foot. The foot is in a markedly plantar flexed position, with the sole facing the adjacent leg in a position of marked adduction. No other anomalies are noted on physical examination. Which of the following is the most likely diagnosis? (A) Epispadias (B) Hypospadias (C) Talipes calcaneovalgus (D) Talipes equinovarus (E) Torticollis
Respuesta: D The correct **answer is D**. This is talipes equinovarus, which is the most common form of clubfoot abnormality. The term talipes is used for any deformity of the foot that involves the talus bone. The term equinus refers to a permanent extension of the foot so that only the ball rests on the ground. The term varus refers to a permanent inversion of the foot, so that only the outer side of the sole touches the ground. Talipes equinovarus is best corrected with repetitive casting of the foot beginning at a young age; this approach slowly brings the foot to a more normal position. Epispadias (**choice A**) and hypospadias (**choice B**) are urethral abnormalities. Talipes calcaneovalgus (**choice C**) is associated with a flat and dorsiflexed foot. Torticollis (**choice E**) is head tilt, which can be present at birth as a result of neck trauma
35
A new mother complains that her 6-week-old infant frequently regurgitates small volumes of formula during and after feedings. Physical examination demonstrates a happy baby who has gained half a pound since his last visit. No abdominal masses are noted. Which of the following is the best next step in management? (A) Change the baby’s formula (B) Change the bottle’s nipple (C) Monitor the baby carefully (D) Order abdominal x-rays (E) Order CT of the abdomen
Respuesta: B The correct **answer is B**. Regurgitation of small volumes (usually less than 5-10 mL) is common among babies and is usually of no clinical significance unless the baby is failing to grow normally. It is of concern to some mothers, and switching to a firmer nipple bottle with a smaller hole will slow the rate that the baby is drinking and often reduce the amount of swallowed air. Changing the baby’s formula (**choice A**) does not usually alter the amount of regurgitation. Careful monitoring of the baby (**choice C**), abdominal x-rays (**choice D**), and CT scan (**choice E**) are not warranted for this benign condition.
36
A 3-year-old girl presents to the pediatrician’s office. The mother states that the girl has been having big, bulky stools that float in the toilet. She also has intermittent diarrhea. On examination, her height is 88 cm (34.6 in, <5th percentile) and weight is 15.8 kg (34.8 lb, <5th percentile). In addition, she has an uncle who died of recurrent lower respiratory infections. Which of the following would be most effective for alleviating the gastrointestinal symptoms of this patient? (A) Avoidance of dairy products (B) Elimination of dietary fat (C) Ketogenic diet (D) Oral metronidazole (E) Pancreatic enzyme replacement
Respuesta: E The correct **answer is E.** This 3-year-old girl has steatorrhea, recurrent diarrhea, and failure to thrive. There is a family history of an uncle who died of recurrent lower respiratory infections. This patient most likely has cystic fibrosis (CF). The gastrointestinal system is significantly affected by CF. Gastrointestinal manifestations are caused by pancreatic exocrine insufficiency, which occurs in more than 85% of patients with CF. The mutation of the CF gene causes inspissation of pancreatic secretions and obstruction of the ductules. It causes the malabsorption of protein and fat, resulting in steatorrhea. Untreated pancreatic insufficiency results in malnutrition and failure to thrive. The treatment is to replace the missing pancreatic enzymes by oral supplementation of purified pancreatic enzymes (pancrelipase) with food. Several capsules are taken with each meal and snack. Even with pancreatic enzyme replacement, however, there might still be mild steatorrhea. Fat-soluble vitamins, such as A, D, E, and K, are also supplemented in these patients to avoid fat-soluble vitamin deficiency. Successful nutritional management is an important part of the overall management of CF, because good nutritional status improves the overall prognosis and decreases the frequency of pulmonary infections. Avoidance of dairy products (**choice A**) is useful in diarrhea caused by lactose intolerance but not in CF. Elimination of dietary fat (**choice B**) is inappropriate. In fact, the intake of fat should be encouraged because the fat content increases palatability and provides high calories to the patient’s body. Dietary fat is also essential for the absorption of fat-soluble vitamins. A ketogenic diet (**choice C**) has no role in the management of CF. It is sometimes used to control epilepsy. Oral metronidazole (**choice D**) is effective against anaerobic bacterial infection. It can be useful in bacterial overgrowth in the intestine and in Clostridium difficile colitis, but it has no role in the treatment of pancreatic insufficiency.
37
A 2-year-old boy presents to the emergency department with fever, irritability, and a skin rash 5 days after the onset of an upper respiratory infection. On examination, his temperature is 39.8 C (103.6 F), and his pulse is 94/ min. There is an erythematous skin rash that involves his face, chest, back, and upper extremities. His skin is very tender to touch. Rubbing the skin causes separation of the epidermal layer. Which of the following is the most likely diagnosis? (A) Kawasaki disease (B) Staphylococcal scalded skin syndrome (C) Streptococcal scarlet fever (D) Toxic epidermal necrolysis (E) Toxic shock syndrome
Respuesta: B The correct **answer is B**. The boy in this clinical vignette has staphylococcal scalded skin syndrome (SSSS). It is characterized by fever, malaise, and a generalized fine erythematous rash. It is then followed by crusty, sometimes exudative, lesions around the mouth and eyes. It is usually preceded by an upper respiratory infection. The skin is tender to touch. With gentle rubbing, the epidermal layer can exfoliate, which is the characteristic Nikolsky sign. This syndrome is caused by an exfoliative toxin secreted by certain strains (55 and 71) of Staphylococcus aureus. Sometimes, injection of the conjunctiva and fissuring of the lips can happen and might be confused with Kawasaki disease. The differential diagnosis includes toxic shock syndrome, scarlet fever, and toxic epidermal necrolysis. Nafcillin or oxacillin can be used to treat SSSS. Kawasaki disease (**choice A**) involves vasculitis of epicardial coronary vessels, with possible development of aneurysm. The clinical syndrome presents with fever longer than 5 days, bilateral conjunctival injection, mucosal change in the oral pharynx, erythema of hands and feet with desquamation, and cervical lymphadenopathy. Transient arthritis may also occur. Streptococcal scarlet fever (**choice C**) usually presents with an erythematous sandpaper-like rash with fever and “strawberry tongue.” It is caused by erythrogenic exotoxins secreted by group A Streptococcus. It is usually associated with streptococcal pharyngitis. Toxic epidermal necrolysis (**choice D**) is a dermatologic emergency characterized by widespread epidermal necrosis and desquamation after formation of blisters and bullae. It is caused by a severe hypersensitivity reaction, usually to a drug. Major complications include sepsis, dehydration, and shock. Patients should be treated as if they have a generalized severe burn. Toxic shock syndrome (**choice E**) is caused by an exotoxin (TSST1) of Staphylococcus aureus, or less commonly, Streptococcus pyogenes. Patients present with high fever, myalgia, pharyngitis, abdominal pain, vomiting, diarrhea, headache, and a diffuse, sunburn-like rash. Severe cases are complicated by hypotension, shock, and renal failure.
38
A frantic mother telephones the pediatric office. She reports that her 10-year-old boy accidentally splashed Drano (a strongly corrosive, alkaline drain cleaner) on his face, and he is screaming in pain complaining that his right eye hurts terribly. Which of the following is the best advice to give to the mother? (A) Apply antibiotic ointment to the eye and make an appointment with an ophthalmologist (B) Bring the boy to the hospital right away (C) Pry the eye open and drip vinegar over it until the pain goes away (D) Pry the eye open and swipe it clean with a tissue before bringing the boy in for further evaluation (E) Pry the eye open, hold it under running cold water for about 30 minutes, and then bring the boy to the hospital
Respuesta: E The correct **answer is E**. Alkaline burns are extremely destructive, and the process of destruction continues as long as the agent is in contact with the tissues. Immediate removal is essential, and the best way to do it is with massive irrigation. In the emergency department, sterile saline would be used; at home, tap water will do. Antibiotics (**choice A**) do not help, and a leisurely approach will guarantee loss of the eye. Rushing to the hospital (**choice B**) might seem like the thing to do for all kinds of emergencies, but in this case immediate first aid is more important. If the corrosive substance remains in the eye during the trip to the emergency department, the eye will be destroyed by the time the patient arrives. "Playing chemist” (**choice C**) is never acceptable because of the possibility of causing an exothermic reaction. Swiping with a tissue (**choice D**) would be better than nothing, but not by much. Swiping has a role, but not as the first thing to do. After massive irrigation, little particulate matter may remain, which may then have to be mechanically swiped away.
39
The mother of a 4-year-old child takes her daughter to a pediatrician because she is “scratching all the time.” Physical examination demonstrates multiple areas of excoriation, which are worst on the shoulders, buttocks, and abdomen. In the areas where the scratching has occurred, scattered tiny red punctate lesions are also seen. Careful examination of the clothing reveals small, ovoid, grayish-white structures attached to threads on the seams. Which of the following is the most likely causative agent? (A) Ancylostoma braziliense (B) Corynebacterium minutissimum (C) Pediculus humanus corporis (D) Sarcoptes scabiei (E) Trichophyton rubrum
Respuesta: C The correct **answer is C**. The child has body lice, Pediculus humanus corporis, which live in the clothing and cause itchy bites. The structures on the clothing seams are the eggs. The lice themselves can also often be seen. The lice are confined to the skin surface, and, unlike scabies mites, do not burrow into the skin. The patient, family members, and social contacts (such as other daycare children) should all be treated at the same time with permethrin cream, 5%. Clothing and bedding should also be washed. Ancylostoma braziliense (**choice A**) causes cutaneous larva migrans, which produces long itchy burrows on the legs and buttocks. Corynebacterium minutissimum (**choice B**) causes erythrasma, characterized by pink to brown itchy patches in moist areas of skin. Sarcoptes scabiei (**choice D**) causes scabies, which is diagnosed by finding fine white lines up to 1 cm in length (burrows) in the skin, with a mite at the end of them. Trichophyton rubrum (**choice E**) is a fungus that can cause ringworm of the body
40
An 8-year-old boy is brought to the emergency department with a head injury. He hit his head on the ground when he fell off his bicycle. He was not wearing a helmet at the time. There was no loss of consciousness. He vomited 2 times after the accident and now complains of a right-sided headache and inability to hear with his right ear. There is no photophobia or diplopia. On physical examination, his blood pressure is 110/72 mm Hg, pulse is 104/min, and respirations are 22/min. He is alert and oriented and responds appropriately to questions. There is a round hematoma on the right side of his head. Bloody drainage is noted from his right ear. Which of the following injury is most consistent with these findings? (A) Concussion (B) Epidural hematoma (C) Subdural hematoma (D) Temporal bone fracture (E) Tympanic membrane perforation
Respuesta: D The correct **answer is D**. Bleeding from the ear following an injury to the skull is pathognomonic of a temporal bone fracture. The bleeding can be medial to an intact tympanic membrane, from the middle ear through a rupture of the tympanic membrane, or from a fracture line in the ear canal. Hemotympanum gives the tympanic membrane a blue-black color. Usually, there is a communication with the subarachnoid space through the fracture line. Often, there is cerebrospinal fluid otorrhea. Cleaning of the ear canal should be avoided for fear of introducing microorganisms. The immediate danger to the patient is the development of meningitis. Prophylactic antibiotic therapy has to be initiated immediately. Most fractures of the temporal bone are longitudinal (80%) to the long axis of the petrous pyramid. Only 20% are transverse. Longitudinal fractures usually extend through the middle ear into the ear canal, causing rupture of the tympanic membrane. Approximately 35% of longitudinal fractures produce a sensorineural hearing loss, and approximately 15% produce facial paralysis. Transverse fractures extend across the cochlea and fallopian canal, causing a profound, permanent sensorineural hearing loss and a facial paralysis. These fractures are usually well demonstrated with CT. Concussion (**choice A**) is a syndrome of alteration of consciousness secondary to head injury, typically brought about by a sudden change in the momentum of the head. Its usual manifestations include loss of consciousness, temporary respiratory arrest, and loss of reflexes. Epidural hematomas (**choice B**) are blood clots that form between the inner table of the skull and the dura. Eighty percent are associated with skull fractures across the middle meningeal artery or across a dural sinus. The high arterial pressure of the bleeding vessel dissects the dura away from the skull, permitting the formation of the hematoma. The incidence of skull fractures in children with epidural hematomas is lower than in adults because of the elasticity of the skull. Subdural hematoma (**choice C**) usually results from trauma involving acceleration or deceleration head injury and is commonly associated with parenchymal brain injury. Tympanic membrane perforation (**choice E**) most commonly happens after an object is inserted into the ear canal and perforates the eardrum.
41
A 16-year-old girl comes to the physician’s office because she has not begun menstruating yet. Both her mother and an older sister started menstruation at age 12. She takes no medication and denies strenuous exercise or excessive dieting. Her height is at the 50th percentile for age; her weight is at the 60th percentile. Both her breast and pubic hair development are at Tanner stage 4. Pelvic ultrasonography reveals a normal uterus and ovaries. Which of the following is the most likely diagnosis? (A) Imperforate hymen (B) Physiologic pubertal delay (C) Prolactinoma (D) Testicular feminization syndrome (E) Turner syndrome
Respuesta: B The correct **answer is B**. Primary amenorrhea is defined as the absence of menses by the age of 16, or 4 years after thelarche (the onset of breast development). The prevalence of primary amenorrhea is 1% to 2%. In this country, the median age of menarche is 12.5 years. Most girls have menarche about 2 years after thelarche. Primary amenorrhea is different from secondary amenorrhea, which is defined as the absence of menses for three menstrual cycles or a maximum of 6 months in women who previously had normal menstruation. Primary amenorrhea has a vast differential diagnosis and is usually divided into three categories: 1) outflow tract anomalies (e.g., imperforate hymen, transverse vaginal septum, vaginal agenesis, testicular feminization); 2) end-organ disorders (e.g., ovarian failure, gonadal agenesis with 46,XY chromosomes); and 3) central disorders (e.g., hypothalamic disorders, pituitary disorders). The age of the adolescent girl in this scenario is consistent with primary amenorrhea. Her sexual maturation is progressing appropriately. It is most likely that she has physiologic pubertal delay because the pelvic ultrasonography is normal. She probably has not completed her pubertal development. Imperforate hymen (**choice A**) does not allow egress of menses. Patients usually present with pelvic pain from the accumulation of menses and dilatation of the uterus. Physical examination reveals a bulging membrane across the introitus and a midline pelvic mass. Prolactinoma (**choice C**) can cause headache, decreased growth rate, galactorrhea, delayed puberty, primary or secondary amenorrhea, gynecomastia, and visual field defects. Testicular feminization syndrome (**choice D**) is caused by either dysfunction or total absence of the testosterone receptor and results in a phenotypical female with 46,XY chromosomes. Patients with testicular feminization will have normal breast development but lack pubic hair. Pelvic ultrasound would reveal agenesis of the vagina and uterus. In Turner syndrome (45,XO; **choice E**), the ovaries undergo rapid atresia; by puberty, there are no primordial oocytes.
42
A 3-week-old infant is being evaluated for hematochezia. His mother states that the infant passed stools that contain both blood and mucus. There were no complications during her pregnancy, and the infant has been otherwise healthy. On physical examination, his temperature is 37.1 C (98.9 F), pulse is 110/min, and respirations are 18/ min. He appears well, and his fontanelle is flat and level. Abdominal examination reveals normal active bowel sounds; his abdomen is nontender to palpation and there is no mass. His diaper contains stool that has bright red blood on it with mucus. Which of the following is the most likely explanation of his hematochezia? (A) Food allergy-induced colitis (B) Meckel diverticulum (C) Necrotizing enterocolitis (D) Rectal fissure (E) Ulcerative colitis
Respuesta: A The correct **answer is A**. Food allergy-induced colitis is a clinical syndrome of enterocolitis usually presenting between 1 week and 3 months of age. The dietary proteins implicated are generally cow’s milk or soy proteins. Although diarrhea and emesis are the most prominent symptoms, rectal bleeding, protein-losing enteropathy with edema, and dehydration with metabolic acidosis are also prominent. The diarrhea is relatively acute and progressive to the point of dehydration if the etiologic protein is not eliminated. The stools contain mucus, polymorphonuclear leukocytes, and eosinophils. Rectal bleeding progresses from occult to increasingly gross bleeding and anemia. Whereas the diarrhea is most consistent with an inflammatory colitis, stools may also contain reducing sugars, suggesting a small intestinal component as well. Approximately one third of infants with severe diarrhea will develop metabolic acidosis. Hypotension may complicate the presentation in younger infants but is more commonly seen following protein challenge, especially with soy. Irritability, emesis, and abdominal distention are also prominent with acute manifestations. When a more indolent clinical course is encountered, the infant will manifest failure to thrive, edema from enteric protein loss, and anemia. Food refusal, a feature of eosinophilic esophagitis and enteropathy, is also common. The diagnosis is confirmed when symptoms resolve within 72-96 hours after the elimination of the offending protein from the diet, and subsequent oral challenge provokes both symptoms and a measurable inflammatory response. More than 90% of the infants will respond to initiation of an extensively hydrolyzed casein-based formula. From 15% to 50% of infants sensitive to cow’s milk will also be sensitive to soy-based formula. Meckel diverticulum (**choice B**) is the most common congenital malformation of the gastrointestinal tract, occurring in about 1.5% of the general population, most of whom experience no symptoms. The mucosa is often gastric, and the acid secreted may cause ulceration and bleeding, which is usually painless. Necrotizing enterocolitis (**choice C**) affects premature infants and usually occurs as a complication of respiratory distress syndrome or patent ductus arteriosus. It manifests as abdominal distention, tenderness, bilious vomiting, and occult or frank blood in stool. Rectal fissure (**choice D**) is one of the most common causes of rectal bleeding in young children. There is usually a history of constipation, with passage of large and hard stools. Ulcerative colitis (**choice E**) is an inflammatory bowel disease primarily affecting the colon. It causes abdominal pain, hematochezia, diarrhea, weight loss, and fever.
43
A 7-year-old boy is brought to the physician because of a persistent mucopurulent nasal discharge for 2 weeks following a common cold. The mother also reports that the child has had frequent cough during the day and occasional temperatures up to 38.0 C (100.5 F). The child does not appear critically ill, but he complains of mild pain in the maxillary region and nasal obstruction. Rhinoscopic examination reveals a rivulet of purulent fluid coming from the inferior meatus. The rest of the physical examination is normal. Which of the following is the most likely diagnosis? (A) Acute bacterial sinusitis (B) Acute otitis media (C) Acute viral rhinitis (D) Allergic rhinosinusitis (E) Asthma
Respuesta: A The correct **answer is A.** Acute otitis media and acute sinusitis are among the most common infectious conditions of childhood. Acute sinusitis usually follows a common cold and is due to bacterial infection of the maxillary sinuses. It manifests with persistent mucopurulent discharge, daytime coughing without wheezing, and nasal obstruction. The diagnosis is based on a temporal criterion, with such manifestations present for at least 10 days. The 10-day mark allows differentiation of acute sinusitis from acute viral rhinitis (**choice C**), which should resolve within a week or so. If nasal discharge and obstruction persist for more than 30 days, the diagnosis is subacute sinusitis. High fever, swelling of eyelids, and pain on slight pressure over the sinuses indicate a severe sinus infection that needs prompt evaluation and aggressive antibiotic treatment. Most cases of acute sinusitis resolve after antibiotic treatment with amoxicillin or a macrolide, plus topical vasoconstrictors and nasal saline lavage for symptomatic relief. Acute otitis media (**choice B**) usually follows an upper respiratory infection. It is characterized by otalgia (especially in older children) and the otoscopic findings of bulging and fullness of the tympanic membrane. Acute otitis media is more frequent between 6 months and 3 years of age. Allergic rhinosinusitis (**choice D**) manifests with chronic clear nasal discharge, without fever or pain. There is usually an accompanying history of allergies. Asthma (**choice E**) is characterized by paroxysmal attacks of respiratory distress with tachypnea and wheezing. Cough and increased nasal secretion may be present
44
A 10-year-old girl comes to medical attention because of recurrent attacks of wheezing and dyspnea. The attacks occur mostly at home or, if outdoors, soon after exercise. Exacerbations are noted in springtime. The severity of symptoms is mild. Pulmonary function tests show that peak expiratory flow (PEF) and forced respiratory volume per second (FEV1 ) are reduced during an attack but are relatively normal during symptom-free intervals. Height and weight are in the 60th percentile. Complete blood count shows 8% eosinophils; all other parameters are normal. Cutaneous testing shows the patient to be allergic to a variety of allergens, including dust mites, animal dander, and several pollens. Which of the following is the most effective step in management? (A) Avoidance of exercise (B) Avoidance of respiratory irritants, such as cigarette smoke (C) Use of a humidifier at home (D) Use of air cleaners at home (E) Administration of multiple-drug regimens (F) Immunotherapy against identified allergens
Respuesta: B The correct **answer is B**. The most crucial step in the management of asthma is avoidance of triggering factors, e.g., allergens. Unfortunately, it is difficult to avoid specific types of allergens, such as pollens. Specific measures to eliminate or reduce exposure to dust mites and animal dander at home lead to reduced frequency of attacks and hospitalization rates. Regardless of the allergens involved, elimination of respiratory irritants, especially cigarette smoke, is of crucial importance. The bronchial tree of asthmatic patients is highly reactive to any form of chemical or physical irritation. Thus, exposure to passive smoke should be absolutely avoided. Avoidance of exercise (**choice A)** is not an appropriate measure, although exercise frequently triggers asthmatic attacks. The efficacy of anti-asthma measures can be evaluated by observing how the child can sustain adequate forms of exercise. Use of a humidifier at home (**choice C**) will favor growth of dust mites and consequently increase the concentration of these allergens in the environment. Use of air cleaners at home (**choice D**) has not been shown to be uniformly effective in getting rid of dust mites. Administration of multiple-drug regimens (**choice E**) is not a recommended strategy in the pharmacologic treatment of asthma. In fact, the fewest number of drugs at the lowest effective doses should be used. Typically, a regimen of one drug (a bronchodilator or a inhaled corticosteroid) for mild-moderate cases or two drugs for more severe cases is sufficient to control asthma exacerbations. Immunotherapy against identified allergens (**choice F**) is of some benefit when a single allergen is involved, but not with multiple airborne allergens.
45
A 3-year-old child is evaluated for possible causes of her mental retardation. Physical examination demonstrates elfin facies. The child has a history of stenosis of the aortic valve and transient hypercalcemia in infancy. Genetic microdeletion studies are performed. The lesion most likely involves which of the following genetic loci? (A) 7q11.23 (B) 20p12 (C) 22q11.21 (D) Maternal chromosome at 15q11 (E) Paternal chromosome at 15q11
Respuesta: A The correct **answer is A**. Mental retardation associated with abnormal facial features may be related to abnormal genetics. As our skill with DNA analysis has improved, we have identified a number of microdeletion syndromes involving contiguous genes on a single chromosome that are associated with mental retardation and often distinctive clinical presentations. The case illustrated is Williams syndrome, which involves 7q11.23. 20p12 (**choice B**) is associated with Alagille syndrome, characterized by lack of bile ducts, cardiac abnormalities, butterfly vertebrae, and eye abnormalities. 22q11.21 (**choice C**) is associated with DiGeorge syndrome, which is characterized by thymus and parathyroid abnormalities, heart defects, cleft palate, and mental retardation. Involvement of the maternal chromosome 15 at 15q11 (**choice D**) is associated with Angelman syndrome, which is characterized by seizures, puppet-like movements, outbursts of laughter, and severe mental retardation. Involvement of the paternal chromosome 15 at 15q11 (**choice E**) is associated with Prader-Willi syndrome, which is characterized by obesity, small hands and feet, and mental retardation.
46
A term neonate is healthy at birth and receives routine perinatal care. The infant is discharged from the hospital on day 3. Ten days after delivery, the infant develops severe erythema and edema in both eyelids. There is an associated watery discharge that soon became copious and mucopurulent, with presence of pseudomembranes. Which of the following conditions is this infant most at risk for? (A) Corneal ulceration (B) Encephalitis (C) Pneumonia (D) Sepsis (E) Silver toxicity
Respuesta: C The correct **answer is C**. This infant has neonatal chlamydial infection that was acquired during birth from an infected mother. The major manifestation presented in this case is chlamydial ophthalmia, which occurs in about 30 to 40% of infants born to infected mothers. The severity of chlamydial ophthalmia can range from mild conjunctivitis to the severe pattern described in the question stem. Follicles are not present in the conjunctiva; they appear in in older children and adults. Chlamydial ophthalmia becomes evident 5 to 14 days after birth. Many of these infants also have nasopharyngeal colonization with Chlamydia, and neonatal pneumonia is common in patients who have clinically developed conjunctivitis. Symptoms of chlamydial pneumonia typically occur within 2 to 19 weeks, with an insidious onset of staccato cough. Gonorrheal ophthalmia produces an acute purulent conjunctivitis that appears 2 to 5 days after birth or earlier with premature rupture of membranes. It can rapidly lead to corneal ulceration (**choice A**) and perforation if treatment is delayed. Serious systemic complications, such as encephalitis (**choice B**), can result from herpes simplex keratocon-junctivits because these infants have poor immunologic response to the generalized herpes infection. Sepsis (**choice D**) can be a complication of conjunctivitis, but it is less specific to chlamydial ophthalmia compared with other types of bacterial conjunctivitis. The risk for chlamydial pneumonia is more significant in this patient. A chemical conjunctivitis may result from prophylactic silver nitrate use in neonates. It usually appears within 6 to 8 hours after installation and disappears spontaneously within 24 to 48 hours. Silver toxicity (**choice E**) is not typical of prophylactic silver nitrate use, and it is unrelated to chlamydial ophthalmia.
47
A 3-month-old, previously well male infant presents to the emergency department in January with a 2-day history of clear rhinorrhea, low-grade fever, and poor appetite, but no cough. On physical examination, there are mild subcostal retractions, coarse breath sounds heard throughout the lung fields, and scattered expiratory wheezes. The child receives an intravenous fluid bolus in the emergency department and is admitted for observation. Which of the following is the most severe, life-threatening complication of this child’s illness? (A) Apnea (B) Congestive heart failure (C) Dehydration (D) Hypoxemia (E) Wheezing
Respuesta: A The correct **answer is A**. Apnea can be a life-threatening complication of RSV infection in infants younger than 1 year. For this reason, infants with respiratory distress and either laboratory confirmation or a clinical picture suggestive of RSV infection are often admitted to the hospital for supportive care and monitoring. Congestive heart failure (**choice B**) is not associated with RSV infection in a child with normal cardiac function. Infants are obligate nose-breathers and may have difficulty feeding because of nasal congestion. In the face of respiratory distress, infants have increased insensible fluid losses and are at risk for dehydration (**choice C**). Dehydration, however, is not the most severe risk for this child. Hypoxemia (**choice D**) and cyanosis may result from mucous plugging of the lower bronchial tree, atelectasis, and mismatch in infants with acute RSV infection. The hypoxemia is usually transient and not life-threatening. Hypoxemia is readily corrected with supplemental oxygen administration. Wheezing (**choice E**) is a very common sign of RSV infection. It signifies inflammation of the bronchioles, but it is not lifethreatening.
48
A 14-year-old boy has pain in his left leg. An x-ray shows a tumor and a biopsy reveals histopathologic features characteristic of neural origin. Which of the following is the most likely diagnosis? (A) Chondroblastoma (B) Ewing sarcoma (C) Neuroblastoma (D) Osteosarcoma (E) Rhabdomyosarcoma
Respuesta: B The correct **answer is B**. Ewing sarcoma (ES) is the second most common bone cancer in childhood and adolescence. It has a peak age incidence during adolescent years. It rarely occurs in blacks and is more prevalent in boys than girls. ES may have histopathologic features characteristic of neural differentiation; ES is thought to be a tumor of neural origin, whereas most bone tumors are thought to be of mesodermal origin. The tumor category of ES has recently been expanded to include more neural tumors, such as peripheral primitive neuroectodermal tumor (PNET). Like other bone tumors, ES is associated with a consistent chromosomal translocation, t(11;22)(q24;q12), which occurs in about 90 to 95% of cases. Chondroblastoma (**choice A**) is a very rare bone tumor that most often occurs in the pelvis. Histopathology reveals cartilaginous tumor tissue with malignant spindle cells. Neuroblastoma (**choice C**) is a common solid tumor of childhood that typically occurs during the first few years of life. Histologically it is a small round cell tumor with characteristic Homer-Wright rosettes. It displays neural differentiation and therefore might initially be confused with ES, but it is not a bone tumor. It usually occurs in the neck, thorax, abdomen, or flank. Osteosarcoma (**choice D**) is the most common bone cancer of childhood and adolescence. Like ES, osteosarcoma may involve the bones of the leg, especially the femur, and frequently metastases to the lungs. Osteosarcoma is known to be associated with certain genetic conditions, such as osteogenic imperfecta, and there is an increased occurrence of osteosarcoma in children with retinoblastoma. It occurs in all races. Pathologically, neural differentiation is not observed. Rhabdomyosarcoma (**choice E**) is the most common soft tissue sarcoma of childhood. It does occur more frequently among males and whites, but it is usually found in the head and neck areas and the genitourinary tract. Like neuroblastoma and ES it is a small round cell tumor. It does not display neural differentiation. It may first be noticed after trauma as in this case, but there is no bony involvement.
49
A 2-year-old child is brought to the emergency department because of generalized convulsions that last 15 minutes. He has had a fever for 24 hours, and his current temperature is 39.5 C (103.0 F). He also has a sore throat, but otherwise looks healthy. His father also had several episodes of febrile seizures in his childhood. Which of the following is the most important factor that will increase the risk of recurrence of febrile seizures? (A) Age older than 18 months (B) Duration of seizure longer than 5 minutes (C) Family history of febrile seizures (D) Fever of long duration before onset of seizure (E) Temperature higher than 39.0 C
Respuesta: C The correct **answer is C**. Febrile seizures are common events. This form of benign manifestation affects approximately 3 to 4% of children. Since epilepsy is defined as at least two seizures occurring without an identifiable trigger, febrile seizures do not constitute epilepsy even if they recur. Fever, in fact, is the identifiable trigger. Most children who have an episode of febrile seizure will not have a recurrence. Risk factors that do increase the likelihood of recurrence include family history, age younger than 18 months (**choice A**), and temperature less than 39.0 C (**choice E**) when seizure occurred. Overall, children with a first febrile seizure will have a recurrence in 25% of cases by 1 year and in 30% by 2 years. The duration of the first seizure (**choice B**) will not influence the likelihood of recurrence, but recurrences will likely also be prolonged if the first episode lasted for more than 15 minutes. Fever of long duration (**choice D**) is negatively correlated to the probability of recurrences.
50
A concerned mother brings her 2-month-old daughter to the clinic because of constipation. The mother had appropriate prenatal care but decided to deliver her child at home with the help of a midwife. The child has not received any medical attention since birth. Examination reveals jaundice, an umbilical hernia, and poor muscle tone. Which of the following is the most appropriate diagnostic study? (A) Alpha-1-antitrypsin genotyping (B) Liver and spleen scan (C) Measurements of T4 and TSH (D) Barium swallow (E) RPR and FTA for syphilis
Respuesta: C The correct **answer is C.** Measurements of T4 and TSH levels are the most appropriate tests to confirm congenital hypothyroidism. Alpha1 antitrypsin genotyping (**choice A**) is used to confirm alpha1 antitrypsin deficiency. A liver and spleen scan (**choice B**) is useful to indicate absence of the biliary tree or blockage of the extrahepatic bile ducts and is therefore most useful to confirm biliary atresia. Barium swallow (**choice D**) will indicate the presence of pyloric stenosis (“string” sign = narrowing of barium stream passing through the duodenum; “umbrella” sign = the hold-up of barium in the stomach). The RPR and FTA (**choice E**) are serologic tests for syphilis.